A conjecture about numbers of the form $10^{m}(2^{k}−1)+2^{k-1}−1$, where $m$ is the number of decimal...












18












$begingroup$


Question




Numbers $n$ of the form $10^{m}(2^{k}−1)+2^{k-1}−1$, where $m$ is the number of decimal digits of $ 2^{k-1}$. For example:





  • $k=1$ then $n=10$.


  • $k=2$ then $n=31$.


  • $k=3$ then $n=73$.


  • $k=4$ then $n=157.$




Conjecture:



the number $(2^k-1)cdot 10^m+2^{k-1}-1$ where $m$ is the number of decimal digits of $2^{k-1}$ is never prime when it is of the form $7s+6$, that is when it is congruent to $6$ $pmod 7$. Examples: $n=1023511$ ($k=10$)$equiv 6 pmod 7$ and thus it is composite $(1023511=19times103times523)$, $n=20471023$ ($k=11$) $equiv 6 pmod 7$ and thus it is composite ($20471023=479times42737)$. With PFGW we arrived to $k=565000$ and all the $n's$ congruent to $6 pmod 7$ are composite. According to Giovanni Resta's calculations in a post which has been canceled, there should be no probable prime congruent to 6 $pmod 7$ upto k=800.000. The residue $6$ $pmod 7$ occurs when either $m=6t+3$ and $k=3l+1$ or $m=6t+4$ and $k=3l+2$ with $k$ and $l$ some non-negative integers, but amazingly when it occurs the number is not prime. Can you find a counter-example or give a proof for the conjecture? Here a link to other interesting questions: Is there a number of the form $f(n)=7k+6=5p$ with prime p? and Why do all residues occur in this similar sequence?
For primes of this form see:
The on-line Encyclopedia of integer sequences
The following vector contains all the exponents k<=366800 leading to a prime



$[2, 3, 4, 7, 8, 12, 19, 22, 36, 46, 51, 67, 79, 215, 359, 394, 451, 1323, 2131, 3336, 3371, 6231, 19179, 39699, 51456, 56238, 69660, 75894, 79798, 92020, 174968, 176006, 181015, 285019, 331259, 360787, 366770]$



Exponent $541456$ leads to another probable prime with residue 5 mod 7 and 325990 digits, but it need not be the next in increasing order.



Remark: we found five-in-a-row probable primes with res 5 mod 7. Probable primes with residue 5 are now twice frequent than expected.
Exponents of these primes seem to be NOT random at all. Another thing I noticed, i don't know if it has some importance: the exponents leading to a probable prime $215, 69660, 92020, 541456$ are multiples of $43$. I noticed that $frac{215}{41}, frac{69660}{41}, frac{92020}{41}, frac{541456}{41}$ all have a periodic decimal expansion equal to $overline{24390}=29^3+1$. This is equivalent to say that when k is a multiple of 43 and the number $10^{m}(2^{k}−1)+2^{k-1}−1$ is prime, then k is of the form $41s+r$ where r is a number in the set (1,10,16,18,37). Is there some mathematical reason for that?










share|cite|improve this question











$endgroup$








  • 2




    $begingroup$
    github.com/gnufinder/special-prime/issues
    $endgroup$
    – Peter
    Feb 22 '18 at 16:56






  • 4




    $begingroup$
    How can you give a bounty of $200$ when you only have $116$ reputation?
    $endgroup$
    – user477343
    Mar 5 '18 at 6:52






  • 1




    $begingroup$
    mathoverflow.net/questions/294527/…
    $endgroup$
    – Peter
    Mar 6 '18 at 12:02






  • 3




    $begingroup$
    It's worth to mention that if $n = (2^k-1)cdot 10^m + 2^{k-1}-1$, then $2n+1=(2cdot 10^m + 1)(2^k - 1)$. In particular, $n$ can never be a Sophie Germain prime.
    $endgroup$
    – Max Alekseyev
    Mar 6 '18 at 17:22








  • 3




    $begingroup$
    Please reduce your editing of this post. It is not necessary to keep the search limit quite up to date in the post. If you want to have the current search limit on the page, post a comment (and delete it when you post the next). Updating the post once or twice a week is plenty enough.
    $endgroup$
    – Daniel Fischer
    Mar 9 '18 at 11:48
















18












$begingroup$


Question




Numbers $n$ of the form $10^{m}(2^{k}−1)+2^{k-1}−1$, where $m$ is the number of decimal digits of $ 2^{k-1}$. For example:





  • $k=1$ then $n=10$.


  • $k=2$ then $n=31$.


  • $k=3$ then $n=73$.


  • $k=4$ then $n=157.$




Conjecture:



the number $(2^k-1)cdot 10^m+2^{k-1}-1$ where $m$ is the number of decimal digits of $2^{k-1}$ is never prime when it is of the form $7s+6$, that is when it is congruent to $6$ $pmod 7$. Examples: $n=1023511$ ($k=10$)$equiv 6 pmod 7$ and thus it is composite $(1023511=19times103times523)$, $n=20471023$ ($k=11$) $equiv 6 pmod 7$ and thus it is composite ($20471023=479times42737)$. With PFGW we arrived to $k=565000$ and all the $n's$ congruent to $6 pmod 7$ are composite. According to Giovanni Resta's calculations in a post which has been canceled, there should be no probable prime congruent to 6 $pmod 7$ upto k=800.000. The residue $6$ $pmod 7$ occurs when either $m=6t+3$ and $k=3l+1$ or $m=6t+4$ and $k=3l+2$ with $k$ and $l$ some non-negative integers, but amazingly when it occurs the number is not prime. Can you find a counter-example or give a proof for the conjecture? Here a link to other interesting questions: Is there a number of the form $f(n)=7k+6=5p$ with prime p? and Why do all residues occur in this similar sequence?
For primes of this form see:
The on-line Encyclopedia of integer sequences
The following vector contains all the exponents k<=366800 leading to a prime



$[2, 3, 4, 7, 8, 12, 19, 22, 36, 46, 51, 67, 79, 215, 359, 394, 451, 1323, 2131, 3336, 3371, 6231, 19179, 39699, 51456, 56238, 69660, 75894, 79798, 92020, 174968, 176006, 181015, 285019, 331259, 360787, 366770]$



Exponent $541456$ leads to another probable prime with residue 5 mod 7 and 325990 digits, but it need not be the next in increasing order.



Remark: we found five-in-a-row probable primes with res 5 mod 7. Probable primes with residue 5 are now twice frequent than expected.
Exponents of these primes seem to be NOT random at all. Another thing I noticed, i don't know if it has some importance: the exponents leading to a probable prime $215, 69660, 92020, 541456$ are multiples of $43$. I noticed that $frac{215}{41}, frac{69660}{41}, frac{92020}{41}, frac{541456}{41}$ all have a periodic decimal expansion equal to $overline{24390}=29^3+1$. This is equivalent to say that when k is a multiple of 43 and the number $10^{m}(2^{k}−1)+2^{k-1}−1$ is prime, then k is of the form $41s+r$ where r is a number in the set (1,10,16,18,37). Is there some mathematical reason for that?










share|cite|improve this question











$endgroup$








  • 2




    $begingroup$
    github.com/gnufinder/special-prime/issues
    $endgroup$
    – Peter
    Feb 22 '18 at 16:56






  • 4




    $begingroup$
    How can you give a bounty of $200$ when you only have $116$ reputation?
    $endgroup$
    – user477343
    Mar 5 '18 at 6:52






  • 1




    $begingroup$
    mathoverflow.net/questions/294527/…
    $endgroup$
    – Peter
    Mar 6 '18 at 12:02






  • 3




    $begingroup$
    It's worth to mention that if $n = (2^k-1)cdot 10^m + 2^{k-1}-1$, then $2n+1=(2cdot 10^m + 1)(2^k - 1)$. In particular, $n$ can never be a Sophie Germain prime.
    $endgroup$
    – Max Alekseyev
    Mar 6 '18 at 17:22








  • 3




    $begingroup$
    Please reduce your editing of this post. It is not necessary to keep the search limit quite up to date in the post. If you want to have the current search limit on the page, post a comment (and delete it when you post the next). Updating the post once or twice a week is plenty enough.
    $endgroup$
    – Daniel Fischer
    Mar 9 '18 at 11:48














18












18








18


16



$begingroup$


Question




Numbers $n$ of the form $10^{m}(2^{k}−1)+2^{k-1}−1$, where $m$ is the number of decimal digits of $ 2^{k-1}$. For example:





  • $k=1$ then $n=10$.


  • $k=2$ then $n=31$.


  • $k=3$ then $n=73$.


  • $k=4$ then $n=157.$




Conjecture:



the number $(2^k-1)cdot 10^m+2^{k-1}-1$ where $m$ is the number of decimal digits of $2^{k-1}$ is never prime when it is of the form $7s+6$, that is when it is congruent to $6$ $pmod 7$. Examples: $n=1023511$ ($k=10$)$equiv 6 pmod 7$ and thus it is composite $(1023511=19times103times523)$, $n=20471023$ ($k=11$) $equiv 6 pmod 7$ and thus it is composite ($20471023=479times42737)$. With PFGW we arrived to $k=565000$ and all the $n's$ congruent to $6 pmod 7$ are composite. According to Giovanni Resta's calculations in a post which has been canceled, there should be no probable prime congruent to 6 $pmod 7$ upto k=800.000. The residue $6$ $pmod 7$ occurs when either $m=6t+3$ and $k=3l+1$ or $m=6t+4$ and $k=3l+2$ with $k$ and $l$ some non-negative integers, but amazingly when it occurs the number is not prime. Can you find a counter-example or give a proof for the conjecture? Here a link to other interesting questions: Is there a number of the form $f(n)=7k+6=5p$ with prime p? and Why do all residues occur in this similar sequence?
For primes of this form see:
The on-line Encyclopedia of integer sequences
The following vector contains all the exponents k<=366800 leading to a prime



$[2, 3, 4, 7, 8, 12, 19, 22, 36, 46, 51, 67, 79, 215, 359, 394, 451, 1323, 2131, 3336, 3371, 6231, 19179, 39699, 51456, 56238, 69660, 75894, 79798, 92020, 174968, 176006, 181015, 285019, 331259, 360787, 366770]$



Exponent $541456$ leads to another probable prime with residue 5 mod 7 and 325990 digits, but it need not be the next in increasing order.



Remark: we found five-in-a-row probable primes with res 5 mod 7. Probable primes with residue 5 are now twice frequent than expected.
Exponents of these primes seem to be NOT random at all. Another thing I noticed, i don't know if it has some importance: the exponents leading to a probable prime $215, 69660, 92020, 541456$ are multiples of $43$. I noticed that $frac{215}{41}, frac{69660}{41}, frac{92020}{41}, frac{541456}{41}$ all have a periodic decimal expansion equal to $overline{24390}=29^3+1$. This is equivalent to say that when k is a multiple of 43 and the number $10^{m}(2^{k}−1)+2^{k-1}−1$ is prime, then k is of the form $41s+r$ where r is a number in the set (1,10,16,18,37). Is there some mathematical reason for that?










share|cite|improve this question











$endgroup$




Question




Numbers $n$ of the form $10^{m}(2^{k}−1)+2^{k-1}−1$, where $m$ is the number of decimal digits of $ 2^{k-1}$. For example:





  • $k=1$ then $n=10$.


  • $k=2$ then $n=31$.


  • $k=3$ then $n=73$.


  • $k=4$ then $n=157.$




Conjecture:



the number $(2^k-1)cdot 10^m+2^{k-1}-1$ where $m$ is the number of decimal digits of $2^{k-1}$ is never prime when it is of the form $7s+6$, that is when it is congruent to $6$ $pmod 7$. Examples: $n=1023511$ ($k=10$)$equiv 6 pmod 7$ and thus it is composite $(1023511=19times103times523)$, $n=20471023$ ($k=11$) $equiv 6 pmod 7$ and thus it is composite ($20471023=479times42737)$. With PFGW we arrived to $k=565000$ and all the $n's$ congruent to $6 pmod 7$ are composite. According to Giovanni Resta's calculations in a post which has been canceled, there should be no probable prime congruent to 6 $pmod 7$ upto k=800.000. The residue $6$ $pmod 7$ occurs when either $m=6t+3$ and $k=3l+1$ or $m=6t+4$ and $k=3l+2$ with $k$ and $l$ some non-negative integers, but amazingly when it occurs the number is not prime. Can you find a counter-example or give a proof for the conjecture? Here a link to other interesting questions: Is there a number of the form $f(n)=7k+6=5p$ with prime p? and Why do all residues occur in this similar sequence?
For primes of this form see:
The on-line Encyclopedia of integer sequences
The following vector contains all the exponents k<=366800 leading to a prime



$[2, 3, 4, 7, 8, 12, 19, 22, 36, 46, 51, 67, 79, 215, 359, 394, 451, 1323, 2131, 3336, 3371, 6231, 19179, 39699, 51456, 56238, 69660, 75894, 79798, 92020, 174968, 176006, 181015, 285019, 331259, 360787, 366770]$



Exponent $541456$ leads to another probable prime with residue 5 mod 7 and 325990 digits, but it need not be the next in increasing order.



Remark: we found five-in-a-row probable primes with res 5 mod 7. Probable primes with residue 5 are now twice frequent than expected.
Exponents of these primes seem to be NOT random at all. Another thing I noticed, i don't know if it has some importance: the exponents leading to a probable prime $215, 69660, 92020, 541456$ are multiples of $43$. I noticed that $frac{215}{41}, frac{69660}{41}, frac{92020}{41}, frac{541456}{41}$ all have a periodic decimal expansion equal to $overline{24390}=29^3+1$. This is equivalent to say that when k is a multiple of 43 and the number $10^{m}(2^{k}−1)+2^{k-1}−1$ is prime, then k is of the form $41s+r$ where r is a number in the set (1,10,16,18,37). Is there some mathematical reason for that?







number-theory integers






share|cite|improve this question















share|cite|improve this question













share|cite|improve this question




share|cite|improve this question








edited Jan 17 at 10:03







Enzo Creti

















asked Feb 4 '18 at 13:13









Enzo CretiEnzo Creti

2041419




2041419








  • 2




    $begingroup$
    github.com/gnufinder/special-prime/issues
    $endgroup$
    – Peter
    Feb 22 '18 at 16:56






  • 4




    $begingroup$
    How can you give a bounty of $200$ when you only have $116$ reputation?
    $endgroup$
    – user477343
    Mar 5 '18 at 6:52






  • 1




    $begingroup$
    mathoverflow.net/questions/294527/…
    $endgroup$
    – Peter
    Mar 6 '18 at 12:02






  • 3




    $begingroup$
    It's worth to mention that if $n = (2^k-1)cdot 10^m + 2^{k-1}-1$, then $2n+1=(2cdot 10^m + 1)(2^k - 1)$. In particular, $n$ can never be a Sophie Germain prime.
    $endgroup$
    – Max Alekseyev
    Mar 6 '18 at 17:22








  • 3




    $begingroup$
    Please reduce your editing of this post. It is not necessary to keep the search limit quite up to date in the post. If you want to have the current search limit on the page, post a comment (and delete it when you post the next). Updating the post once or twice a week is plenty enough.
    $endgroup$
    – Daniel Fischer
    Mar 9 '18 at 11:48














  • 2




    $begingroup$
    github.com/gnufinder/special-prime/issues
    $endgroup$
    – Peter
    Feb 22 '18 at 16:56






  • 4




    $begingroup$
    How can you give a bounty of $200$ when you only have $116$ reputation?
    $endgroup$
    – user477343
    Mar 5 '18 at 6:52






  • 1




    $begingroup$
    mathoverflow.net/questions/294527/…
    $endgroup$
    – Peter
    Mar 6 '18 at 12:02






  • 3




    $begingroup$
    It's worth to mention that if $n = (2^k-1)cdot 10^m + 2^{k-1}-1$, then $2n+1=(2cdot 10^m + 1)(2^k - 1)$. In particular, $n$ can never be a Sophie Germain prime.
    $endgroup$
    – Max Alekseyev
    Mar 6 '18 at 17:22








  • 3




    $begingroup$
    Please reduce your editing of this post. It is not necessary to keep the search limit quite up to date in the post. If you want to have the current search limit on the page, post a comment (and delete it when you post the next). Updating the post once or twice a week is plenty enough.
    $endgroup$
    – Daniel Fischer
    Mar 9 '18 at 11:48








2




2




$begingroup$
github.com/gnufinder/special-prime/issues
$endgroup$
– Peter
Feb 22 '18 at 16:56




$begingroup$
github.com/gnufinder/special-prime/issues
$endgroup$
– Peter
Feb 22 '18 at 16:56




4




4




$begingroup$
How can you give a bounty of $200$ when you only have $116$ reputation?
$endgroup$
– user477343
Mar 5 '18 at 6:52




$begingroup$
How can you give a bounty of $200$ when you only have $116$ reputation?
$endgroup$
– user477343
Mar 5 '18 at 6:52




1




1




$begingroup$
mathoverflow.net/questions/294527/…
$endgroup$
– Peter
Mar 6 '18 at 12:02




$begingroup$
mathoverflow.net/questions/294527/…
$endgroup$
– Peter
Mar 6 '18 at 12:02




3




3




$begingroup$
It's worth to mention that if $n = (2^k-1)cdot 10^m + 2^{k-1}-1$, then $2n+1=(2cdot 10^m + 1)(2^k - 1)$. In particular, $n$ can never be a Sophie Germain prime.
$endgroup$
– Max Alekseyev
Mar 6 '18 at 17:22






$begingroup$
It's worth to mention that if $n = (2^k-1)cdot 10^m + 2^{k-1}-1$, then $2n+1=(2cdot 10^m + 1)(2^k - 1)$. In particular, $n$ can never be a Sophie Germain prime.
$endgroup$
– Max Alekseyev
Mar 6 '18 at 17:22






3




3




$begingroup$
Please reduce your editing of this post. It is not necessary to keep the search limit quite up to date in the post. If you want to have the current search limit on the page, post a comment (and delete it when you post the next). Updating the post once or twice a week is plenty enough.
$endgroup$
– Daniel Fischer
Mar 9 '18 at 11:48




$begingroup$
Please reduce your editing of this post. It is not necessary to keep the search limit quite up to date in the post. If you want to have the current search limit on the page, post a comment (and delete it when you post the next). Updating the post once or twice a week is plenty enough.
$endgroup$
– Daniel Fischer
Mar 9 '18 at 11:48










2 Answers
2






active

oldest

votes


















8












$begingroup$

According to your list, a counter-example, if it exists, must have more than $60,000$ digits. So, a counterexample would be a quite gigantic prime.



Unfortunately, a proof of the conjecture will almost certainly be out of reach.



The search for a counter-example can be painful as well, it is well possible that the smallest is already too big for current algorithms for primality testing.






share|cite|improve this answer











$endgroup$









  • 2




    $begingroup$
    @EnzoCreti No, this is only known for arithmetic progressions of the form $an+b$ with coprime $a$ and $b$ (Dirichlet). The numbers here will behave like "random" numbers , hence we won't be able to rule out a large prime.
    $endgroup$
    – Peter
    Feb 5 '18 at 12:33






  • 3




    $begingroup$
    @Peter Every number is of the form $10^{m}(2^k-1)+2^{k-1}-1$, where $m$ is the number of decimal digits of $2^{k-1}$. As $log_{2}10$ is irrational, the residues of $m$ and $k$ modulo 6, and hence the residues of $10^{m}$ and $2^{k-1}$ modulo 7, are independent. You can find out the distribution simply by tabulating the possible cases.
    $endgroup$
    – Taneli Huuskonen
    Feb 6 '18 at 16:03








  • 3




    $begingroup$
    My observation just explains why different residues modulo 7 occur at different frequencies.
    $endgroup$
    – Taneli Huuskonen
    Feb 6 '18 at 19:34






  • 2




    $begingroup$
    math.stackexchange.com/questions/2658464/…
    $endgroup$
    – Peter
    Feb 20 '18 at 18:51






  • 2




    $begingroup$
    For $k=32$, we have the factorization $$131cdot 4463cdot 21601cdot 44623 cdot 76213$$ This is a counterexample with residue $6$
    $endgroup$
    – Peter
    Feb 21 '18 at 10:57





















1












$begingroup$

@peter told me about this and i found it very interesting.

I made this tool, so anybody can help computing. Simply download and run to participate. Just refresh the page to update stats.

With 30-40 person, getting to $n=10^7$ should not be to long.






share|cite|improve this answer









$endgroup$













  • $begingroup$
    How many people participated already ?
    $endgroup$
    – Peter
    Aug 3 '18 at 18:46










  • $begingroup$
    @DanaJ have you a routine for Sage whcih allows to test this conjecture?
    $endgroup$
    – Enzo Creti
    Dec 3 '18 at 13:45











Your Answer





StackExchange.ifUsing("editor", function () {
return StackExchange.using("mathjaxEditing", function () {
StackExchange.MarkdownEditor.creationCallbacks.add(function (editor, postfix) {
StackExchange.mathjaxEditing.prepareWmdForMathJax(editor, postfix, [["$", "$"], ["\\(","\\)"]]);
});
});
}, "mathjax-editing");

StackExchange.ready(function() {
var channelOptions = {
tags: "".split(" "),
id: "69"
};
initTagRenderer("".split(" "), "".split(" "), channelOptions);

StackExchange.using("externalEditor", function() {
// Have to fire editor after snippets, if snippets enabled
if (StackExchange.settings.snippets.snippetsEnabled) {
StackExchange.using("snippets", function() {
createEditor();
});
}
else {
createEditor();
}
});

function createEditor() {
StackExchange.prepareEditor({
heartbeatType: 'answer',
autoActivateHeartbeat: false,
convertImagesToLinks: true,
noModals: true,
showLowRepImageUploadWarning: true,
reputationToPostImages: 10,
bindNavPrevention: true,
postfix: "",
imageUploader: {
brandingHtml: "Powered by u003ca class="icon-imgur-white" href="https://imgur.com/"u003eu003c/au003e",
contentPolicyHtml: "User contributions licensed under u003ca href="https://creativecommons.org/licenses/by-sa/3.0/"u003ecc by-sa 3.0 with attribution requiredu003c/au003e u003ca href="https://stackoverflow.com/legal/content-policy"u003e(content policy)u003c/au003e",
allowUrls: true
},
noCode: true, onDemand: true,
discardSelector: ".discard-answer"
,immediatelyShowMarkdownHelp:true
});


}
});














draft saved

draft discarded


















StackExchange.ready(
function () {
StackExchange.openid.initPostLogin('.new-post-login', 'https%3a%2f%2fmath.stackexchange.com%2fquestions%2f2635516%2fa-conjecture-about-numbers-of-the-form-10m2k%25e2%2588%259212k-1%25e2%2588%25921-where-m-is%23new-answer', 'question_page');
}
);

Post as a guest















Required, but never shown

























2 Answers
2






active

oldest

votes








2 Answers
2






active

oldest

votes









active

oldest

votes






active

oldest

votes









8












$begingroup$

According to your list, a counter-example, if it exists, must have more than $60,000$ digits. So, a counterexample would be a quite gigantic prime.



Unfortunately, a proof of the conjecture will almost certainly be out of reach.



The search for a counter-example can be painful as well, it is well possible that the smallest is already too big for current algorithms for primality testing.






share|cite|improve this answer











$endgroup$









  • 2




    $begingroup$
    @EnzoCreti No, this is only known for arithmetic progressions of the form $an+b$ with coprime $a$ and $b$ (Dirichlet). The numbers here will behave like "random" numbers , hence we won't be able to rule out a large prime.
    $endgroup$
    – Peter
    Feb 5 '18 at 12:33






  • 3




    $begingroup$
    @Peter Every number is of the form $10^{m}(2^k-1)+2^{k-1}-1$, where $m$ is the number of decimal digits of $2^{k-1}$. As $log_{2}10$ is irrational, the residues of $m$ and $k$ modulo 6, and hence the residues of $10^{m}$ and $2^{k-1}$ modulo 7, are independent. You can find out the distribution simply by tabulating the possible cases.
    $endgroup$
    – Taneli Huuskonen
    Feb 6 '18 at 16:03








  • 3




    $begingroup$
    My observation just explains why different residues modulo 7 occur at different frequencies.
    $endgroup$
    – Taneli Huuskonen
    Feb 6 '18 at 19:34






  • 2




    $begingroup$
    math.stackexchange.com/questions/2658464/…
    $endgroup$
    – Peter
    Feb 20 '18 at 18:51






  • 2




    $begingroup$
    For $k=32$, we have the factorization $$131cdot 4463cdot 21601cdot 44623 cdot 76213$$ This is a counterexample with residue $6$
    $endgroup$
    – Peter
    Feb 21 '18 at 10:57


















8












$begingroup$

According to your list, a counter-example, if it exists, must have more than $60,000$ digits. So, a counterexample would be a quite gigantic prime.



Unfortunately, a proof of the conjecture will almost certainly be out of reach.



The search for a counter-example can be painful as well, it is well possible that the smallest is already too big for current algorithms for primality testing.






share|cite|improve this answer











$endgroup$









  • 2




    $begingroup$
    @EnzoCreti No, this is only known for arithmetic progressions of the form $an+b$ with coprime $a$ and $b$ (Dirichlet). The numbers here will behave like "random" numbers , hence we won't be able to rule out a large prime.
    $endgroup$
    – Peter
    Feb 5 '18 at 12:33






  • 3




    $begingroup$
    @Peter Every number is of the form $10^{m}(2^k-1)+2^{k-1}-1$, where $m$ is the number of decimal digits of $2^{k-1}$. As $log_{2}10$ is irrational, the residues of $m$ and $k$ modulo 6, and hence the residues of $10^{m}$ and $2^{k-1}$ modulo 7, are independent. You can find out the distribution simply by tabulating the possible cases.
    $endgroup$
    – Taneli Huuskonen
    Feb 6 '18 at 16:03








  • 3




    $begingroup$
    My observation just explains why different residues modulo 7 occur at different frequencies.
    $endgroup$
    – Taneli Huuskonen
    Feb 6 '18 at 19:34






  • 2




    $begingroup$
    math.stackexchange.com/questions/2658464/…
    $endgroup$
    – Peter
    Feb 20 '18 at 18:51






  • 2




    $begingroup$
    For $k=32$, we have the factorization $$131cdot 4463cdot 21601cdot 44623 cdot 76213$$ This is a counterexample with residue $6$
    $endgroup$
    – Peter
    Feb 21 '18 at 10:57
















8












8








8





$begingroup$

According to your list, a counter-example, if it exists, must have more than $60,000$ digits. So, a counterexample would be a quite gigantic prime.



Unfortunately, a proof of the conjecture will almost certainly be out of reach.



The search for a counter-example can be painful as well, it is well possible that the smallest is already too big for current algorithms for primality testing.






share|cite|improve this answer











$endgroup$



According to your list, a counter-example, if it exists, must have more than $60,000$ digits. So, a counterexample would be a quite gigantic prime.



Unfortunately, a proof of the conjecture will almost certainly be out of reach.



The search for a counter-example can be painful as well, it is well possible that the smallest is already too big for current algorithms for primality testing.







share|cite|improve this answer














share|cite|improve this answer



share|cite|improve this answer








edited Feb 19 '18 at 20:26









editor XD

54




54










answered Feb 4 '18 at 22:05









PeterPeter

47.6k1039131




47.6k1039131








  • 2




    $begingroup$
    @EnzoCreti No, this is only known for arithmetic progressions of the form $an+b$ with coprime $a$ and $b$ (Dirichlet). The numbers here will behave like "random" numbers , hence we won't be able to rule out a large prime.
    $endgroup$
    – Peter
    Feb 5 '18 at 12:33






  • 3




    $begingroup$
    @Peter Every number is of the form $10^{m}(2^k-1)+2^{k-1}-1$, where $m$ is the number of decimal digits of $2^{k-1}$. As $log_{2}10$ is irrational, the residues of $m$ and $k$ modulo 6, and hence the residues of $10^{m}$ and $2^{k-1}$ modulo 7, are independent. You can find out the distribution simply by tabulating the possible cases.
    $endgroup$
    – Taneli Huuskonen
    Feb 6 '18 at 16:03








  • 3




    $begingroup$
    My observation just explains why different residues modulo 7 occur at different frequencies.
    $endgroup$
    – Taneli Huuskonen
    Feb 6 '18 at 19:34






  • 2




    $begingroup$
    math.stackexchange.com/questions/2658464/…
    $endgroup$
    – Peter
    Feb 20 '18 at 18:51






  • 2




    $begingroup$
    For $k=32$, we have the factorization $$131cdot 4463cdot 21601cdot 44623 cdot 76213$$ This is a counterexample with residue $6$
    $endgroup$
    – Peter
    Feb 21 '18 at 10:57
















  • 2




    $begingroup$
    @EnzoCreti No, this is only known for arithmetic progressions of the form $an+b$ with coprime $a$ and $b$ (Dirichlet). The numbers here will behave like "random" numbers , hence we won't be able to rule out a large prime.
    $endgroup$
    – Peter
    Feb 5 '18 at 12:33






  • 3




    $begingroup$
    @Peter Every number is of the form $10^{m}(2^k-1)+2^{k-1}-1$, where $m$ is the number of decimal digits of $2^{k-1}$. As $log_{2}10$ is irrational, the residues of $m$ and $k$ modulo 6, and hence the residues of $10^{m}$ and $2^{k-1}$ modulo 7, are independent. You can find out the distribution simply by tabulating the possible cases.
    $endgroup$
    – Taneli Huuskonen
    Feb 6 '18 at 16:03








  • 3




    $begingroup$
    My observation just explains why different residues modulo 7 occur at different frequencies.
    $endgroup$
    – Taneli Huuskonen
    Feb 6 '18 at 19:34






  • 2




    $begingroup$
    math.stackexchange.com/questions/2658464/…
    $endgroup$
    – Peter
    Feb 20 '18 at 18:51






  • 2




    $begingroup$
    For $k=32$, we have the factorization $$131cdot 4463cdot 21601cdot 44623 cdot 76213$$ This is a counterexample with residue $6$
    $endgroup$
    – Peter
    Feb 21 '18 at 10:57










2




2




$begingroup$
@EnzoCreti No, this is only known for arithmetic progressions of the form $an+b$ with coprime $a$ and $b$ (Dirichlet). The numbers here will behave like "random" numbers , hence we won't be able to rule out a large prime.
$endgroup$
– Peter
Feb 5 '18 at 12:33




$begingroup$
@EnzoCreti No, this is only known for arithmetic progressions of the form $an+b$ with coprime $a$ and $b$ (Dirichlet). The numbers here will behave like "random" numbers , hence we won't be able to rule out a large prime.
$endgroup$
– Peter
Feb 5 '18 at 12:33




3




3




$begingroup$
@Peter Every number is of the form $10^{m}(2^k-1)+2^{k-1}-1$, where $m$ is the number of decimal digits of $2^{k-1}$. As $log_{2}10$ is irrational, the residues of $m$ and $k$ modulo 6, and hence the residues of $10^{m}$ and $2^{k-1}$ modulo 7, are independent. You can find out the distribution simply by tabulating the possible cases.
$endgroup$
– Taneli Huuskonen
Feb 6 '18 at 16:03






$begingroup$
@Peter Every number is of the form $10^{m}(2^k-1)+2^{k-1}-1$, where $m$ is the number of decimal digits of $2^{k-1}$. As $log_{2}10$ is irrational, the residues of $m$ and $k$ modulo 6, and hence the residues of $10^{m}$ and $2^{k-1}$ modulo 7, are independent. You can find out the distribution simply by tabulating the possible cases.
$endgroup$
– Taneli Huuskonen
Feb 6 '18 at 16:03






3




3




$begingroup$
My observation just explains why different residues modulo 7 occur at different frequencies.
$endgroup$
– Taneli Huuskonen
Feb 6 '18 at 19:34




$begingroup$
My observation just explains why different residues modulo 7 occur at different frequencies.
$endgroup$
– Taneli Huuskonen
Feb 6 '18 at 19:34




2




2




$begingroup$
math.stackexchange.com/questions/2658464/…
$endgroup$
– Peter
Feb 20 '18 at 18:51




$begingroup$
math.stackexchange.com/questions/2658464/…
$endgroup$
– Peter
Feb 20 '18 at 18:51




2




2




$begingroup$
For $k=32$, we have the factorization $$131cdot 4463cdot 21601cdot 44623 cdot 76213$$ This is a counterexample with residue $6$
$endgroup$
– Peter
Feb 21 '18 at 10:57






$begingroup$
For $k=32$, we have the factorization $$131cdot 4463cdot 21601cdot 44623 cdot 76213$$ This is a counterexample with residue $6$
$endgroup$
– Peter
Feb 21 '18 at 10:57













1












$begingroup$

@peter told me about this and i found it very interesting.

I made this tool, so anybody can help computing. Simply download and run to participate. Just refresh the page to update stats.

With 30-40 person, getting to $n=10^7$ should not be to long.






share|cite|improve this answer









$endgroup$













  • $begingroup$
    How many people participated already ?
    $endgroup$
    – Peter
    Aug 3 '18 at 18:46










  • $begingroup$
    @DanaJ have you a routine for Sage whcih allows to test this conjecture?
    $endgroup$
    – Enzo Creti
    Dec 3 '18 at 13:45
















1












$begingroup$

@peter told me about this and i found it very interesting.

I made this tool, so anybody can help computing. Simply download and run to participate. Just refresh the page to update stats.

With 30-40 person, getting to $n=10^7$ should not be to long.






share|cite|improve this answer









$endgroup$













  • $begingroup$
    How many people participated already ?
    $endgroup$
    – Peter
    Aug 3 '18 at 18:46










  • $begingroup$
    @DanaJ have you a routine for Sage whcih allows to test this conjecture?
    $endgroup$
    – Enzo Creti
    Dec 3 '18 at 13:45














1












1








1





$begingroup$

@peter told me about this and i found it very interesting.

I made this tool, so anybody can help computing. Simply download and run to participate. Just refresh the page to update stats.

With 30-40 person, getting to $n=10^7$ should not be to long.






share|cite|improve this answer









$endgroup$



@peter told me about this and i found it very interesting.

I made this tool, so anybody can help computing. Simply download and run to participate. Just refresh the page to update stats.

With 30-40 person, getting to $n=10^7$ should not be to long.







share|cite|improve this answer












share|cite|improve this answer



share|cite|improve this answer










answered Jul 21 '18 at 22:54









François HuppéFrançois Huppé

362111




362111












  • $begingroup$
    How many people participated already ?
    $endgroup$
    – Peter
    Aug 3 '18 at 18:46










  • $begingroup$
    @DanaJ have you a routine for Sage whcih allows to test this conjecture?
    $endgroup$
    – Enzo Creti
    Dec 3 '18 at 13:45


















  • $begingroup$
    How many people participated already ?
    $endgroup$
    – Peter
    Aug 3 '18 at 18:46










  • $begingroup$
    @DanaJ have you a routine for Sage whcih allows to test this conjecture?
    $endgroup$
    – Enzo Creti
    Dec 3 '18 at 13:45
















$begingroup$
How many people participated already ?
$endgroup$
– Peter
Aug 3 '18 at 18:46




$begingroup$
How many people participated already ?
$endgroup$
– Peter
Aug 3 '18 at 18:46












$begingroup$
@DanaJ have you a routine for Sage whcih allows to test this conjecture?
$endgroup$
– Enzo Creti
Dec 3 '18 at 13:45




$begingroup$
@DanaJ have you a routine for Sage whcih allows to test this conjecture?
$endgroup$
– Enzo Creti
Dec 3 '18 at 13:45


















draft saved

draft discarded




















































Thanks for contributing an answer to Mathematics Stack Exchange!


  • Please be sure to answer the question. Provide details and share your research!

But avoid



  • Asking for help, clarification, or responding to other answers.

  • Making statements based on opinion; back them up with references or personal experience.


Use MathJax to format equations. MathJax reference.


To learn more, see our tips on writing great answers.




draft saved


draft discarded














StackExchange.ready(
function () {
StackExchange.openid.initPostLogin('.new-post-login', 'https%3a%2f%2fmath.stackexchange.com%2fquestions%2f2635516%2fa-conjecture-about-numbers-of-the-form-10m2k%25e2%2588%259212k-1%25e2%2588%25921-where-m-is%23new-answer', 'question_page');
}
);

Post as a guest















Required, but never shown





















































Required, but never shown














Required, but never shown












Required, but never shown







Required, but never shown

































Required, but never shown














Required, but never shown












Required, but never shown







Required, but never shown







Popular posts from this blog

Le Mesnil-Réaume

Ida-Boy-Ed-Garten

web3.py web3.isConnected() returns false always